- PowerScore Staff
- Posts: 5972
- Joined: Mar 25, 2011
- Thu Jul 05, 2018 5:36 pm
#47516
Complete Question Explanation
(The complete setup for this game can be found here: lsat/viewtopic.php?t=9394)
The correct answer choice is (A)
The sixth song in the sequence is a ballad, and so the answer cannot be (C), (D), or (E). If the third song in the sequence is S, then V must be fifth. From the second rule H must be played earlier in the sequence than V, and so H cannot be sixth. Thus, answer choice (B) can be eliminated, and answer choice (A) must be the correct answer.
(The complete setup for this game can be found here: lsat/viewtopic.php?t=9394)
The correct answer choice is (A)
The sixth song in the sequence is a ballad, and so the answer cannot be (C), (D), or (E). If the third song in the sequence is S, then V must be fifth. From the second rule H must be played earlier in the sequence than V, and so H cannot be sixth. Thus, answer choice (B) can be eliminated, and answer choice (A) must be the correct answer.
Dave Killoran
PowerScore Test Preparation
Follow me on X/Twitter at http://twitter.com/DaveKilloran
My LSAT Articles: http://blog.powerscore.com/lsat/author/dave-killoran
PowerScore Podcast: http://www.powerscore.com/lsat/podcast/
PowerScore Test Preparation
Follow me on X/Twitter at http://twitter.com/DaveKilloran
My LSAT Articles: http://blog.powerscore.com/lsat/author/dave-killoran
PowerScore Podcast: http://www.powerscore.com/lsat/podcast/